LSAT and Law School Admissions Forum

Get expert LSAT preparation and law school admissions advice from PowerScore Test Preparation.

User avatar
 Dave Killoran
PowerScore Staff
  • PowerScore Staff
  • Posts: 5848
  • Joined: Mar 25, 2011
|
#94747
Complete Question Explanation
(The complete setup for this game can be found here: lsat/viewtopic.php?f=269&t=1940)

The correct answer choice is (B)

The condition in the question stem specifies that L stocks aisles 1 and 9:

G1-Q7-d1.png

Of course, the fourth rule regarding J not stocking consecutive aisles is still in force, and given the remaining unplaced variables above, is likely to play a role in this question.

If O stocks aisle 6, then J would have to stock aisles 7 and 8, causing a violation of the fourth rule. Thus, answer choice (B) cannot be true and is correct.
 SherryZ
  • Posts: 124
  • Joined: Oct 06, 2013
|
#11981
Q7: I chose C. But the right answer is B.

I did not pick B because I made this sequence:

J, K, M, K, M, O, J, L, L
1 2 3 4 5 6 7 8 9

Could you tell me what the problem is?

Thank you very much!

----Sherry
User avatar
 Dave Killoran
PowerScore Staff
  • PowerScore Staff
  • Posts: 5848
  • Joined: Mar 25, 2011
|
#11987
Hi Sherry,

The problem with your hypothetical above is that it violates the condition in the question stem which states that "Larisa stocks both end aisles."

Thanks!

Get the most out of your LSAT Prep Plus subscription.

Analyze and track your performance with our Testing and Analytics Package.